A 65-year-old man presents to your office for evaluation of valvular heart disease. He is asymptomatic. He walks 5 miles a day without difficulty. An echocardiogram reveals severe AS, with a maximum aortic jet velocity of 4.7 m/s by Doppler echocardiography. LV systolic function is preserved. There is mild LV hypertrophy (wall thickness 1.4 cm). He walks on a treadmill for 9 minutes, with a normal hemodynamic response.
What is the likelihood that he will become symptomatic, or come to surgery, within the next 3 years?
>50%. He has a velocity across the aortic valve of >4 m/s. Observational studies would suggest a high likelihood of symptom development in the next 3 years.
A 52-year-old man who previously underwent AVR with a tilting disk valve presents to you several months following a documented transient ischemic attack (TIA). He has no symptoms at present. Workup at the time of his TIA included carotid Dopplers, and transthoracic and transesophageal echocardiogram. These were unremarkable. The valve was well seated and was functioning normally. No thrombus was seen. Only minimal aortic atheroma was seen. No intracardiac shunt was identified. He has been on warfarin throughout and has maintained an INR between 2 and 3. INR was 2.2 at the time of his TIA. On examination, he is in no acute distress. BP is 120/80 mmHg; pulse is 68 and regular. Carotid upstrokes are full and not delayed. Crisp valve closure sound is heard along with a short, early-peaking systolic ejection murmur at the base. No S3 is heard. P2 is normal. No peripheral edema is noted.
Which of the following would you recommend?
Start ASA, 81 mg/day, and increase warfarin to achieve an INR of 2.5 to 3.5. ACC/AHA guidelines recommended an INR of 2.5 to 3.5 for patients with bileaflet tilting disk mechanical valves in the aortic position who have had a thromboembolic event, and who have atrial fibrillation, LV dysfunction, or a hypercoagulable state. Addition of low-dose ASA (75 to 100 mg) is a class I indication for all patients with mechanical heart valves, and those with the above risk factors and bioprosthetic valves.
If his transesophageal study had revealed a small (1 to 2 mm) echodensity on the valve strut—suggestive of thrombus—but no obstruction to valve function, what should have been done?
Intravenous heparin. Since a small clot was present (without any obstruction to valve function) the patient would benefit from increased anticoagulant therapy. If he were to fail this, then the other alternatives could be considered, such as continuous infusion thrombolytic therapy. Such continuous therapy (although not bolus) could also be used as primary treatment. No established indications exist at this time for glycoprotein IIb/IIIa inhibitors in this clinical setting. It would not be advisable to proceed to reoperation just yet, in the absence of a large clot burden or obstruction to inflow. If treatment with heparin leads to clot resolution, then subsequent warfarin dosing should be increased to maintain INR in the 3.0 to 4.0 range.
You are following a 50-year-old man with moderate mitral stenosis, who had been asymptomatic. He presents to the emergency room with complaints of mild exertional dyspnea and palpitations, present for the past 3 to 4 days. On arrival, he appears comfortable, with an O2 saturation of 99% on room air. His pulse rate is 140 bpm and irregular. BP is 130/75 mmHg. Electrocardiogram reveals atrial fibrillation.
The patient spontaneously converts to sinus rhythm.
Which of the following are you most likely to recommend?
Therapy with warfarin. He should be on warfarin. Valvuloplasty in this setting (onset of atrial fibrillation, in an otherwise asymptomatic patient) is a class IIb indication by current guidelines.
A 34-year-old woman presents to your office for evaluation because she had been on treatment with anorectic agents 5 years ago. She is asymptomatic at this time. She is now at her ideal body weight. On examination, she is in no acute distress. BP is 107/68 mmHg. Jugular venous pulsations appear normal. Chest is clear. Cardiac examination reveals a nondisplaced PMI. S1 and S2 are normal, with an appropriate physiologic split of S2 . P2 is not loud. No S3 or S4 is heard. Auscultation is performed with the patient sitting, supine, and in the left lateral decubitus position. No murmur is heard.
What do you most likely recommend for this patient?
Reassurance, with a repeat physical examination in 6 months. If a thorough physical examination reveals no signs of cardiopulmonary disease and the patient has no symptoms, then reassurance and follow-up are all that are required. Echocardiography should be performed if obesity limits the physical examination or if signs or symptoms are present.